Continuity of $sum_{ngeq 0} a_n x^n$ on $(-1,1)$












1












$begingroup$


If the series $sum_{ngeq 0} a_n$ converges, does it follow that the power series $sum_{ngeq 0} a_n x^n$ converge to a continuous function on $(-1,1)$?



Will showing uniform convergence of $sum_{ngeq 0} a_n x^n$ help?










share|cite|improve this question









$endgroup$












  • $begingroup$
    Yes and yes. I am quite sure you can easily prove uniform convergence on $[-r,r]$, $1>r >0$. And from Abel’s theorem (iirc) the series is defined and left continuous at $x=1$.
    $endgroup$
    – Mindlack
    Jan 26 at 23:28






  • 1




    $begingroup$
    @Mindlack Abel says that if it defined on 1, it is left continuous there
    $endgroup$
    – Holo
    Jan 26 at 23:30










  • $begingroup$
    @Mindlack I don't know of Abel's theorem. Can it be proved in some other way?
    $endgroup$
    – user397197
    Jan 26 at 23:31
















1












$begingroup$


If the series $sum_{ngeq 0} a_n$ converges, does it follow that the power series $sum_{ngeq 0} a_n x^n$ converge to a continuous function on $(-1,1)$?



Will showing uniform convergence of $sum_{ngeq 0} a_n x^n$ help?










share|cite|improve this question









$endgroup$












  • $begingroup$
    Yes and yes. I am quite sure you can easily prove uniform convergence on $[-r,r]$, $1>r >0$. And from Abel’s theorem (iirc) the series is defined and left continuous at $x=1$.
    $endgroup$
    – Mindlack
    Jan 26 at 23:28






  • 1




    $begingroup$
    @Mindlack Abel says that if it defined on 1, it is left continuous there
    $endgroup$
    – Holo
    Jan 26 at 23:30










  • $begingroup$
    @Mindlack I don't know of Abel's theorem. Can it be proved in some other way?
    $endgroup$
    – user397197
    Jan 26 at 23:31














1












1








1





$begingroup$


If the series $sum_{ngeq 0} a_n$ converges, does it follow that the power series $sum_{ngeq 0} a_n x^n$ converge to a continuous function on $(-1,1)$?



Will showing uniform convergence of $sum_{ngeq 0} a_n x^n$ help?










share|cite|improve this question









$endgroup$




If the series $sum_{ngeq 0} a_n$ converges, does it follow that the power series $sum_{ngeq 0} a_n x^n$ converge to a continuous function on $(-1,1)$?



Will showing uniform convergence of $sum_{ngeq 0} a_n x^n$ help?







real-analysis continuity power-series






share|cite|improve this question













share|cite|improve this question











share|cite|improve this question




share|cite|improve this question










asked Jan 26 at 23:25







user397197



















  • $begingroup$
    Yes and yes. I am quite sure you can easily prove uniform convergence on $[-r,r]$, $1>r >0$. And from Abel’s theorem (iirc) the series is defined and left continuous at $x=1$.
    $endgroup$
    – Mindlack
    Jan 26 at 23:28






  • 1




    $begingroup$
    @Mindlack Abel says that if it defined on 1, it is left continuous there
    $endgroup$
    – Holo
    Jan 26 at 23:30










  • $begingroup$
    @Mindlack I don't know of Abel's theorem. Can it be proved in some other way?
    $endgroup$
    – user397197
    Jan 26 at 23:31


















  • $begingroup$
    Yes and yes. I am quite sure you can easily prove uniform convergence on $[-r,r]$, $1>r >0$. And from Abel’s theorem (iirc) the series is defined and left continuous at $x=1$.
    $endgroup$
    – Mindlack
    Jan 26 at 23:28






  • 1




    $begingroup$
    @Mindlack Abel says that if it defined on 1, it is left continuous there
    $endgroup$
    – Holo
    Jan 26 at 23:30










  • $begingroup$
    @Mindlack I don't know of Abel's theorem. Can it be proved in some other way?
    $endgroup$
    – user397197
    Jan 26 at 23:31
















$begingroup$
Yes and yes. I am quite sure you can easily prove uniform convergence on $[-r,r]$, $1>r >0$. And from Abel’s theorem (iirc) the series is defined and left continuous at $x=1$.
$endgroup$
– Mindlack
Jan 26 at 23:28




$begingroup$
Yes and yes. I am quite sure you can easily prove uniform convergence on $[-r,r]$, $1>r >0$. And from Abel’s theorem (iirc) the series is defined and left continuous at $x=1$.
$endgroup$
– Mindlack
Jan 26 at 23:28




1




1




$begingroup$
@Mindlack Abel says that if it defined on 1, it is left continuous there
$endgroup$
– Holo
Jan 26 at 23:30




$begingroup$
@Mindlack Abel says that if it defined on 1, it is left continuous there
$endgroup$
– Holo
Jan 26 at 23:30












$begingroup$
@Mindlack I don't know of Abel's theorem. Can it be proved in some other way?
$endgroup$
– user397197
Jan 26 at 23:31




$begingroup$
@Mindlack I don't know of Abel's theorem. Can it be proved in some other way?
$endgroup$
– user397197
Jan 26 at 23:31










1 Answer
1






active

oldest

votes


















3












$begingroup$

For $|x| leq 1-epsilon$ the series $sum |a_n| |x^{n}|$ is dominated by the convergent series $C(1-epsilon)^{n}$ where $C=sup ]{|a_n|:ngeq 1}$. By M-test, the given series converges uniformly for $|x| leq 1-epsilon$ and hence its sum is continuous there. Since $epsilon $ is arbitrary we are done. Note that $C <infty$ because $a_n to 0$.






share|cite|improve this answer











$endgroup$













  • $begingroup$
    But we do not have that $a_n$ is absolutely convergent
    $endgroup$
    – user397197
    Jan 26 at 23:33












  • $begingroup$
    He doesn't use that $sum a_n$ converges absolutely anywhere (instead, he proves that $sum a_nx^n$ converges absolutely and uniformly for $|x|leq1-varepsilon$).
    $endgroup$
    – Clayton
    Jan 26 at 23:34












  • $begingroup$
    @Matematleta The question has nothing to do with convergence at $x=-1$.
    $endgroup$
    – Kavi Rama Murthy
    Jan 26 at 23:52










  • $begingroup$
    Yes, I misunderstood the question. I deleted my comment. I thought the OP was asking about convergence at the boundary. The more interesting fact is that if $sum a_n$ converges then $f(x)to sum a_n$ as $xto 1.$
    $endgroup$
    – Matematleta
    Jan 26 at 23:54













Your Answer





StackExchange.ifUsing("editor", function () {
return StackExchange.using("mathjaxEditing", function () {
StackExchange.MarkdownEditor.creationCallbacks.add(function (editor, postfix) {
StackExchange.mathjaxEditing.prepareWmdForMathJax(editor, postfix, [["$", "$"], ["\\(","\\)"]]);
});
});
}, "mathjax-editing");

StackExchange.ready(function() {
var channelOptions = {
tags: "".split(" "),
id: "69"
};
initTagRenderer("".split(" "), "".split(" "), channelOptions);

StackExchange.using("externalEditor", function() {
// Have to fire editor after snippets, if snippets enabled
if (StackExchange.settings.snippets.snippetsEnabled) {
StackExchange.using("snippets", function() {
createEditor();
});
}
else {
createEditor();
}
});

function createEditor() {
StackExchange.prepareEditor({
heartbeatType: 'answer',
autoActivateHeartbeat: false,
convertImagesToLinks: true,
noModals: true,
showLowRepImageUploadWarning: true,
reputationToPostImages: 10,
bindNavPrevention: true,
postfix: "",
imageUploader: {
brandingHtml: "Powered by u003ca class="icon-imgur-white" href="https://imgur.com/"u003eu003c/au003e",
contentPolicyHtml: "User contributions licensed under u003ca href="https://creativecommons.org/licenses/by-sa/3.0/"u003ecc by-sa 3.0 with attribution requiredu003c/au003e u003ca href="https://stackoverflow.com/legal/content-policy"u003e(content policy)u003c/au003e",
allowUrls: true
},
noCode: true, onDemand: true,
discardSelector: ".discard-answer"
,immediatelyShowMarkdownHelp:true
});


}
});














draft saved

draft discarded


















StackExchange.ready(
function () {
StackExchange.openid.initPostLogin('.new-post-login', 'https%3a%2f%2fmath.stackexchange.com%2fquestions%2f3088917%2fcontinuity-of-sum-n-geq-0-a-n-xn-on-1-1%23new-answer', 'question_page');
}
);

Post as a guest















Required, but never shown
























1 Answer
1






active

oldest

votes








1 Answer
1






active

oldest

votes









active

oldest

votes






active

oldest

votes









3












$begingroup$

For $|x| leq 1-epsilon$ the series $sum |a_n| |x^{n}|$ is dominated by the convergent series $C(1-epsilon)^{n}$ where $C=sup ]{|a_n|:ngeq 1}$. By M-test, the given series converges uniformly for $|x| leq 1-epsilon$ and hence its sum is continuous there. Since $epsilon $ is arbitrary we are done. Note that $C <infty$ because $a_n to 0$.






share|cite|improve this answer











$endgroup$













  • $begingroup$
    But we do not have that $a_n$ is absolutely convergent
    $endgroup$
    – user397197
    Jan 26 at 23:33












  • $begingroup$
    He doesn't use that $sum a_n$ converges absolutely anywhere (instead, he proves that $sum a_nx^n$ converges absolutely and uniformly for $|x|leq1-varepsilon$).
    $endgroup$
    – Clayton
    Jan 26 at 23:34












  • $begingroup$
    @Matematleta The question has nothing to do with convergence at $x=-1$.
    $endgroup$
    – Kavi Rama Murthy
    Jan 26 at 23:52










  • $begingroup$
    Yes, I misunderstood the question. I deleted my comment. I thought the OP was asking about convergence at the boundary. The more interesting fact is that if $sum a_n$ converges then $f(x)to sum a_n$ as $xto 1.$
    $endgroup$
    – Matematleta
    Jan 26 at 23:54


















3












$begingroup$

For $|x| leq 1-epsilon$ the series $sum |a_n| |x^{n}|$ is dominated by the convergent series $C(1-epsilon)^{n}$ where $C=sup ]{|a_n|:ngeq 1}$. By M-test, the given series converges uniformly for $|x| leq 1-epsilon$ and hence its sum is continuous there. Since $epsilon $ is arbitrary we are done. Note that $C <infty$ because $a_n to 0$.






share|cite|improve this answer











$endgroup$













  • $begingroup$
    But we do not have that $a_n$ is absolutely convergent
    $endgroup$
    – user397197
    Jan 26 at 23:33












  • $begingroup$
    He doesn't use that $sum a_n$ converges absolutely anywhere (instead, he proves that $sum a_nx^n$ converges absolutely and uniformly for $|x|leq1-varepsilon$).
    $endgroup$
    – Clayton
    Jan 26 at 23:34












  • $begingroup$
    @Matematleta The question has nothing to do with convergence at $x=-1$.
    $endgroup$
    – Kavi Rama Murthy
    Jan 26 at 23:52










  • $begingroup$
    Yes, I misunderstood the question. I deleted my comment. I thought the OP was asking about convergence at the boundary. The more interesting fact is that if $sum a_n$ converges then $f(x)to sum a_n$ as $xto 1.$
    $endgroup$
    – Matematleta
    Jan 26 at 23:54
















3












3








3





$begingroup$

For $|x| leq 1-epsilon$ the series $sum |a_n| |x^{n}|$ is dominated by the convergent series $C(1-epsilon)^{n}$ where $C=sup ]{|a_n|:ngeq 1}$. By M-test, the given series converges uniformly for $|x| leq 1-epsilon$ and hence its sum is continuous there. Since $epsilon $ is arbitrary we are done. Note that $C <infty$ because $a_n to 0$.






share|cite|improve this answer











$endgroup$



For $|x| leq 1-epsilon$ the series $sum |a_n| |x^{n}|$ is dominated by the convergent series $C(1-epsilon)^{n}$ where $C=sup ]{|a_n|:ngeq 1}$. By M-test, the given series converges uniformly for $|x| leq 1-epsilon$ and hence its sum is continuous there. Since $epsilon $ is arbitrary we are done. Note that $C <infty$ because $a_n to 0$.







share|cite|improve this answer














share|cite|improve this answer



share|cite|improve this answer








edited Jan 26 at 23:35

























answered Jan 26 at 23:32









Kavi Rama MurthyKavi Rama Murthy

69.2k53169




69.2k53169












  • $begingroup$
    But we do not have that $a_n$ is absolutely convergent
    $endgroup$
    – user397197
    Jan 26 at 23:33












  • $begingroup$
    He doesn't use that $sum a_n$ converges absolutely anywhere (instead, he proves that $sum a_nx^n$ converges absolutely and uniformly for $|x|leq1-varepsilon$).
    $endgroup$
    – Clayton
    Jan 26 at 23:34












  • $begingroup$
    @Matematleta The question has nothing to do with convergence at $x=-1$.
    $endgroup$
    – Kavi Rama Murthy
    Jan 26 at 23:52










  • $begingroup$
    Yes, I misunderstood the question. I deleted my comment. I thought the OP was asking about convergence at the boundary. The more interesting fact is that if $sum a_n$ converges then $f(x)to sum a_n$ as $xto 1.$
    $endgroup$
    – Matematleta
    Jan 26 at 23:54




















  • $begingroup$
    But we do not have that $a_n$ is absolutely convergent
    $endgroup$
    – user397197
    Jan 26 at 23:33












  • $begingroup$
    He doesn't use that $sum a_n$ converges absolutely anywhere (instead, he proves that $sum a_nx^n$ converges absolutely and uniformly for $|x|leq1-varepsilon$).
    $endgroup$
    – Clayton
    Jan 26 at 23:34












  • $begingroup$
    @Matematleta The question has nothing to do with convergence at $x=-1$.
    $endgroup$
    – Kavi Rama Murthy
    Jan 26 at 23:52










  • $begingroup$
    Yes, I misunderstood the question. I deleted my comment. I thought the OP was asking about convergence at the boundary. The more interesting fact is that if $sum a_n$ converges then $f(x)to sum a_n$ as $xto 1.$
    $endgroup$
    – Matematleta
    Jan 26 at 23:54


















$begingroup$
But we do not have that $a_n$ is absolutely convergent
$endgroup$
– user397197
Jan 26 at 23:33






$begingroup$
But we do not have that $a_n$ is absolutely convergent
$endgroup$
– user397197
Jan 26 at 23:33














$begingroup$
He doesn't use that $sum a_n$ converges absolutely anywhere (instead, he proves that $sum a_nx^n$ converges absolutely and uniformly for $|x|leq1-varepsilon$).
$endgroup$
– Clayton
Jan 26 at 23:34






$begingroup$
He doesn't use that $sum a_n$ converges absolutely anywhere (instead, he proves that $sum a_nx^n$ converges absolutely and uniformly for $|x|leq1-varepsilon$).
$endgroup$
– Clayton
Jan 26 at 23:34














$begingroup$
@Matematleta The question has nothing to do with convergence at $x=-1$.
$endgroup$
– Kavi Rama Murthy
Jan 26 at 23:52




$begingroup$
@Matematleta The question has nothing to do with convergence at $x=-1$.
$endgroup$
– Kavi Rama Murthy
Jan 26 at 23:52












$begingroup$
Yes, I misunderstood the question. I deleted my comment. I thought the OP was asking about convergence at the boundary. The more interesting fact is that if $sum a_n$ converges then $f(x)to sum a_n$ as $xto 1.$
$endgroup$
– Matematleta
Jan 26 at 23:54






$begingroup$
Yes, I misunderstood the question. I deleted my comment. I thought the OP was asking about convergence at the boundary. The more interesting fact is that if $sum a_n$ converges then $f(x)to sum a_n$ as $xto 1.$
$endgroup$
– Matematleta
Jan 26 at 23:54




















draft saved

draft discarded




















































Thanks for contributing an answer to Mathematics Stack Exchange!


  • Please be sure to answer the question. Provide details and share your research!

But avoid



  • Asking for help, clarification, or responding to other answers.

  • Making statements based on opinion; back them up with references or personal experience.


Use MathJax to format equations. MathJax reference.


To learn more, see our tips on writing great answers.




draft saved


draft discarded














StackExchange.ready(
function () {
StackExchange.openid.initPostLogin('.new-post-login', 'https%3a%2f%2fmath.stackexchange.com%2fquestions%2f3088917%2fcontinuity-of-sum-n-geq-0-a-n-xn-on-1-1%23new-answer', 'question_page');
}
);

Post as a guest















Required, but never shown





















































Required, but never shown














Required, but never shown












Required, but never shown







Required, but never shown

































Required, but never shown














Required, but never shown












Required, but never shown







Required, but never shown







Popular posts from this blog

Can a sorcerer learn a 5th-level spell early by creating spell slots using the Font of Magic feature?

Does disintegrating a polymorphed enemy still kill it after the 2018 errata?

A Topological Invariant for $pi_3(U(n))$